Noether’s theorem -- Question about symmetry coordinate transformation

In summary, the conversation discusses the concept of symmetry in coordinate transformations and its application in Noether's theorem. The speaker asks about the arbitrary nature of εf(t) and how it affects the symmetry of the new path. They also inquire about the proof of δS = 0 in terms of εf(t) and Noether's theorem. The expert suggests dividing Noether's theorem into two parts and explains that the general case can be reduced to the first part through a change of time.
  • #1
gionole
281
24
TL;DR Summary
Question about symmetry coordinate transformation
Let's say we got one particle in x direction only and we got some motion x(t) which we figured it out through Lagrangian.

In Noether's theorem for coordinate transformation symmetry, we start with the following:

x(t)' = x(t) +εf(t) (ε - some number) - I denoted new path with x(t)'

I'm focusing now on εf(t). In Noether's theorem, εf(t) can't really be any arbitrary function(note that for 2 particles, it must be the same, but I'm not asking that). The reason it can't be arbitrary is if it is kind of wiggling function, then adding it to x(t) will result in a x(t)' where every coordinate of the old path didn't move by the same distance to result in the new path.

Q1: Is the εf(t) really arbitrary and can be anything ?(I know it's small, but that doesn't mean it can't be such a function where middle of it is much bigger than left one, which could cause the new path trajectory definitely not symmetrically moved from old one). Namely, I thought what we mean by coordinate transformation is moving every bit of point on the current trajectory by the same amount. I'm asking as I haven't seen such restrictions in textbooks and if it can be anything, then trajectory has not moved symmetrically - while in Lagrangian, it can be any wiggling function.

Could you say/explain where I'm making wrong statements ? what would you feel like I'm missing ? The best way would be to follow the single particle example first.

Q2: If every point on the trajectory didn't move by the same distance because of εf(t), then we wouldn't have δS = 0 right ? How do I prove that ONLY the symmetric movement of the whole trajectory would cause δS = 0 ? If you assume that δS = 0, then you can get the momentum conservation, but I'm more interested in the proof of δS = 0 in terms of εf(t) and Noether's theorem.
 
Last edited:
Physics news on Phys.org
  • #2
Well, let's divide Noether into parts.

1) ##L(x,\dot x)=L\Big(g^s(x),\frac{\partial g^s}{\partial x}\dot x\Big),\quad \forall s,##
here ##g^s## is a group of symmetry generated by a vector field ##v(x)##. In this case, Noether says that there exists a first integral of the Lagrangian system:
$$I(x,\dot x)=\frac{\partial L}{\partial \dot x}v.$$
Direct calculation checks it.
2) The general case when the Lagrangian depends on t and the symmetry group is defined on the space (t,x) is reduced to 1) by the following trick.

Assume that ##L=L(t,x,\dot x)##; and perform a change of time ##t=t(\tau)##. From the Least Action Principle, it follows that;
in this new time, the system is described by the following autonomous Lagrangian
$$\tilde L(t,x,x',t')=L(t,x,x'/t')t',\quad '=\frac{d}{d\tau}$$
 
Last edited:
  • #3
wrobel said:
Well, let's divide Noether into parts.

1) ##L(x,\dot x)=L\Big(g^s(x),\frac{\partial g^s}{\partial x}\dot x\Big),\quad \forall s,##
here ##g^s## is a group of symmetry generated by a vector field ##v(x)##. In this case, Noether says that there exists a first integral of the Lagrangian system:
$$I(x,\dot x)=\frac{\partial L}{\partial \dot x}v.$$
Direct calculation checks it.
2) The general case when both the Lagrangian and the symmetry group depend on the time is reduced to 1) by the following trick.

Assume that ##L=L(t,x,\dot x)##; and perform a change of time ##t=t(\tau)##. From the Least Action Principle, it follows that;
in this new time, the system is described by the following autonomous Lagrangian
$$\tilde L(t,x,x',t')=L(t(\tau),x,x'/t')t',\quad '=\frac{d}{d\tau}$$
I am sorry but this does not help me as I really cant imagine what g and s are even though you said it. Could you read my question and explain it in my words ? Thank you
 
  • #4
gionole said:
explain it in my words ?
Your words are harder to understand than mine, especially the hardness increases due to relaxed language common for physics textbooks.
Good luck!
 
Last edited:
  • #5
gionole said:
TL;DR Summary: Question about symmetry coordinate transformation

Let's say we got one particle in x direction only and we got some motion x(t) which we figured it out through Lagrangian.

In Noether's theorem for coordinate transformation symmetry, we start with the following:

x(t)' = x(t) +εf(t) (ε - some number) - I denoted new path with x(t)'

I'm focusing now on εf(t). In Noether's theorem, εf(t) can't really be any arbitrary function(note that for 2 particles, it must be the same, but I'm not asking that). The reason it can't be arbitrary is if it is kind of wiggling function, then adding it to x(t) will result in a x(t)' where every coordinate of the old path didn't move by the same distance to result in the new path.

Q1: Is the εf(t) really arbitrary and can be anything ?(I know it's small, but that doesn't mean it can't be such a function where middle of it is much bigger than left one, which could cause the new path trajectory definitely not symmetrically moved from old one). Namely, I thought what we mean by coordinate transformation is moving every bit of point on the current trajectory by the same amount. I'm asking as I haven't seen such restrictions in textbooks and if it can be anything, then trajectory has not moved symmetrically - while in Lagrangian, it can be any wiggling function.

Could you say/explain where I'm making wrong statements ? what would you feel like I'm missing ? The best way would be to follow the single particle example first.

Q2: If every point on the trajectory didn't move by the same distance because of εf(t), then we wouldn't have δS = 0 right ? How do I prove that ONLY the symmetric movement of the whole trajectory would cause δS = 0 ? If you assume that δS = 0, then you can get the momentum conservation, but I'm more interested in the proof of δS = 0 in terms of εf(t) and Noether's theorem.
In Noether's theorem it's sufficient to consider "infinitesimal transformations" of time and configuration-space variables. In the Lagrange formalism it's of the form
$$q^{\prime j}=q^j + \delta \epsilon Q^j(q,\dot{q},t), \quad t'=t+\delta \epsilon \Theta(q,\dot{q},t).$$
Then you demand that the new Lagrangian is equivalent to the old, i.e., that there exists a function ##\Omega(q^j,t)## (it must NOT be dependent on the ##\dot{q}^j##!)
$$L'(q',\dot{q}',t) = \frac{\mathrm{d} t'}{\mathrm{d} t} L[q^{\prime j}(q,\dot{q},t),\dot{q}^{\prime j}(q,\dot{q},t)) + \frac{\mathrm{d}}{\mathrm{d} t} \Omega(q^j,t).$$
Then the above infinitesimal transformation is a symmetry transformation.

Then, using the equations of motion (Euler-Lagrange equations) you can show that this implies the existence of a conserved quantity, which is given in terms of ##Q^j##, ##T##, and ##\Omega##.

I think there's a good treatment in Scheck's textbook on mechanics:

F. Scheck, Mechanics, Springer (2010)
 

1. What is Noether’s theorem?

Noether’s theorem is a fundamental principle in physics that relates symmetries in physical systems to conservation laws. It was first developed by mathematician Emmy Noether in 1915.

2. What is the significance of Noether’s theorem?

Noether’s theorem is significant because it provides a powerful tool for understanding the underlying symmetries in physical systems and their corresponding conservation laws. This allows for a deeper understanding of the laws of nature and can lead to new insights and discoveries in physics.

3. How does Noether’s theorem relate to coordinate transformations?

Noether’s theorem states that for every continuous symmetry in a physical system, there exists a corresponding conservation law. Coordinate transformations are a type of symmetry, and Noether’s theorem shows that they are connected to the conservation of energy, momentum, and angular momentum.

4. Can Noether’s theorem be applied to all physical systems?

Yes, Noether’s theorem can be applied to all physical systems as long as they exhibit some form of symmetry. This includes classical and quantum systems, as well as systems in both the macroscopic and microscopic scales.

5. Are there any limitations to Noether’s theorem?

One limitation of Noether’s theorem is that it only applies to continuous symmetries. It cannot be applied to discrete symmetries, such as the symmetry of a crystal lattice. Additionally, Noether’s theorem does not provide a way to determine the exact form of the conserved quantity, only that it exists.

Similar threads

Replies
11
Views
1K
Replies
3
Views
944
Replies
7
Views
774
Replies
2
Views
191
Replies
4
Views
864
  • Mechanics
Replies
13
Views
987
Replies
5
Views
880
Replies
25
Views
1K
  • Mechanics
Replies
14
Views
1K
  • Mechanics
Replies
3
Views
974
Back
Top